LSAT and Law School Admissions Forum

Get expert LSAT preparation and law school admissions advice from PowerScore Test Preparation.

User avatar
 Dave Killoran
PowerScore Staff
  • PowerScore Staff
  • Posts: 5853
  • Joined: Mar 25, 2011
|
#26437
Complete Question Explanation
(The complete setup for this game can be found here: lsat/viewtopic.php?t=11009)

The correct answer choice is (E)

Because P received more votes than O, and because O received more votes than K, we can infer that P received more votes than K. Therefore, answer choice (E) is correct.
 lhatz
  • Posts: 4
  • Joined: Sep 19, 2016
|
#28752
Can’t answer A also be true, since J comes before O and N has to come after O therefore J needs to come before N
 Adam Tyson
PowerScore Staff
  • PowerScore Staff
  • Posts: 5153
  • Joined: Apr 14, 2011
|
#28771
Answer A does set out a possible order, lhatz, but pay careful attention to the question stem here. You are to select an answer that MUST be true, not one that merely CAN be true, and J doesn't have to be before N (the order could, for example, be LPNJOKM). Careful reading of the question stem will help avoid those kinds of mistakes.
 karunyavgopal
  • Posts: 3
  • Joined: Dec 29, 2016
|
#31663
For must be true questions in pure sequencing games, does it make sense to immediately eliminate answer choices that suggest a relationship between 2 variables on different levels? For example, in this question, any answers that state that Luck or Nipi must have more or less votes than any name other than Ping (except for a possible answer choice that states that Nipi must have more votes than Mist)?
 Kristina Moen
PowerScore Staff
  • PowerScore Staff
  • Posts: 230
  • Joined: Nov 17, 2016
|
#31675
Hi Karunyav,

Since this a Pure Sequencing game, you can rely exclusively on your setup to answer this question. Careful with the concept of different "levels." Let's take a look at the setup for this game:

Image

Luck and Nipi are still on the same "level" as O, K, and M. This could easily have been written with LPOKM in a straight line with N branched off of P and J branched off of O. It doesn't matter what "level" the variables are on - what matters is how the variables are connected to it. For example, I can go in a line from L-P-O-K-M, but I can not go in a line from L-P-J because J is branched off of O in the wrong direction. However, I could go in a line from J-O-K-M.

The fastest way to approach this question is to look at each answer choice and ask yourself "does it have to be true?" from your diagram. For example, I approach Answer Choice (A) and ask "Does J have to come sometime before N?" Nope, N branches off of P and J branches off of O. Sounds like I'm doing something similar to what you are describing - however, be careful thinking of these as "levels." For example, N does have to come sometime after L, even though they might appear to be on different "levels" in the above diagram.

Hope this helps.
 vweber
  • Posts: 1
  • Joined: Jan 04, 2017
|
#31888
In the set up explanation it says that "either L or J" must have received the most votes. Doesn't this mean that L or J are the only two that can sit in places 1 and 2? And if so, and this is why I am confused, aren't both A and E MBT? (I know they aren't, simply writing out my thought process).

Thanks!
Veronica
 Adam Tyson
PowerScore Staff
  • PowerScore Staff
  • Posts: 5153
  • Joined: Apr 14, 2011
|
#31901
Hey Veronica, thanks for asking! It looks like you are making an intuitive leap that isn't justified by the rules. L and J are the only two that are eligible to be first, but that is not the same as saying they must be the first two in line. That would be like saying that Donald and Hillary are the only two who can be elected President, so whichever one is not elected President must be Vice President! That would be a weird world, wouldn't it? (History fans take note and feel free to comment).

The reason only J or L can be first is because they are the only two in our diagram that do not HAVE to have something before them. Nothing is to their left in their respective branches. However, once you place one of them first, other options may open up. With L first, J could be second, but so could P. J could be third, but so could N. J cannot be later than fourth, because it comes before O-K-M, but it doesn't have to be before N, as illustrated earlier in this thread.

So, while answer A could be true, it is not a correct answer in this Must Be True question. Only answer E must be true because of the sequence of P-O-K.

Practice with those pure sequencing diagrams! That type of game is still a player in the modern LSAT, as shown by unusual variation of one found on the September 2016 test.

Good luck!
 blj1228
  • Posts: 1
  • Joined: Mar 10, 2017
|
#33357
Hello,

I do not understand how answer B and A are wrong and how answer E is correct.

I knew answer C and D were wrong based on the initial information stated in the stimulus for the games.


thank you,

B
 Robert Carroll
PowerScore Staff
  • PowerScore Staff
  • Posts: 1787
  • Joined: Dec 06, 2013
|
#33363
B,

Because this is a Global Must Be True question, we should be able to answer the question based on the initial main diagram.

Looking at the diagram Kristina posted above, you can see there is no relation between J and N. Thus, any particular relation between the two does not need to be true, so answer choice (A) does not have to be true.

There is also no relation between K and N, so answer choice (B) does not have to be true.

There is a relation between P and K:

P :longline: O :longline: K

Because the order can be chained, P must receive more votes than K, as answer choice (E) says.

Robert Carroll

Get the most out of your LSAT Prep Plus subscription.

Analyze and track your performance with our Testing and Analytics Package.